ChaseDream

标题: [已解决] GWD-23-Q39 [打印本页]

作者: miffycn    时间: 2007-5-2 14:11
标题: [已解决] GWD-23-Q39

41.: GWD-23-Q39
In Gandania, where the government has a monopoly on tobacco sales, the incidence of smoking-related health problems has risen steadily for the last twenty years.  The health secretary recently proposed a series of laws aimed at curtailing tobacco use in Gandania.  Profits from tobacco sales, however, account for ten percent of Gandania’s annual revenues.  Therefore, Gandania cannot afford to institute the proposed laws.

Which of the following, if true, most seriously weakens the argument?

1All health care in Gandania is government-funded.
2Implementing the proposed laws is not likely to cause a significant increase in the amount of tobacco Gandania exports.
3The percentage of revenue Gandania receives from tobacco sales has remained steady in recent years.
4Profits from tobacco sales far surpass any other single source of revenue for the Gandanian government.
5No government official in Gandania has ever previously proposed laws aimed at curtailing tobacco use.

答案选A我选B,不太明白 请NN explain !!


[此贴子已经被作者于2007-5-3 9:27:31编辑过]

作者: miffycn    时间: 2007-5-2 22:13
没有人回答....
作者: gonghao    时间: 2007-5-2 23:02

When the governement of G pay for the health problem itself, the governement is still getting the profit from its tobacco sales.

When the governement of G save money from the health problem , the governement is still lossing the profit from its tobacco sales.

So the answer is A.

B is the result of implementing the policy with increse of the sales in tobacco that would never happen. And it makes no sense on the conlusion.


作者: miffycn    时间: 2007-5-3 09:27
谢谢NN~~~!!!赞~~~!!!
作者: laura466    时间: 2009-7-28 22:04
up




欢迎光临 ChaseDream (https://forum.chasedream.com/) Powered by Discuz! X3.3